Solving tricky derivative help!

Sota Vik

New member
Joined
Oct 20, 2020
Messages
2
3BC3E2AB-DF02-4ECB-A81B-7A0CD5877BC9.jpeg
I have been trying to solve this derivative for a couple hours now to no avail! Any help would be appreciated.
 
\(\displaystyle y= (ln(x))^{sec(x)}\) so \(\displaystyle ln(y)= sec(x)ln(ln(x))\).

Differentiating that, \(\displaystyle \frac{1}{y}\frac{dy}{dx}= [sec(x)tan(x)]ln(ln(x))+ sec(x)\frac{1}{ln(x)}\frac{1}{x}\)....................corrected

So \(\displaystyle \frac{dy}{dx}= y([sec(x)tan(x)]ln(ln(x))+ sec(x)\frac{1}{(ln(x))}\frac{1}{x})\) ....................corrected

\(\displaystyle = (ln(x))^{sec(x)}([sec(x)tan(x)]ln(ln(x))+ sec(x)\frac{1}{ln((x))}\frac{1}{x})\).....................corrected
 
To OP, Prof Halls made a slight mistake. Starting with the 2nd line where ever you see ln(ln(x)) it should be ln(x).
 
To OP, Prof Halls made a slight mistake. Starting with the 2nd line where ever you see ln(ln(x)) it should be ln(x).
Please write (post) the corrected version you propose.
 
To OP, Prof Halls made a slight mistake. Starting with the 2nd line where ever you see ln(ln(x)) it should be ln(x).

I half agree with Jomo :)

Differentiating that, \(\displaystyle \frac{1}{y}\frac{dy}{dx}= [sec(x)tan(x)]ln(ln(x))+ sec(x)\frac{1}{ln(ln(x))}\frac{1}{x}\)

HallsofIvy got it mostly correct but I think there's a slight mistake (corrected now)shown by the red highlight below (note the first ln(ln(x)) is correct)...

\(\displaystyle \frac{1}{y}\frac{dy}{dx}= [sec(x)tan(x)]ln(ln(x))+ sec(x)\frac{1}{\color{red}ln(ln(x))\color{black}}\frac{1}{x}\)

So this line ought to be...

\(\displaystyle \frac{1}{y}\frac{dy}{dx}= [\sec(x)\tan(x)]\ln(\ln(x))+ \sec(x)\frac{1}{\ln(x)}\frac{1}{x}\)

Continuing to the final answer, simplify

\(\displaystyle = \sec(x)\left(\tan(x)\ln(\ln(x))+ \frac{1}{x\ln(x)}\right)\)

Multiply both sides by y

\(\displaystyle \frac{dy}{dx} =\left[ \ln(x)\right]^{sec(x)}\sec(x)\left(\tan(x)\ln(\ln(x))+ \frac{1}{x\ln(x)}\right) \)

Wolfram drew a graph that looks correct, but their symbolic result seems incorrect because I plotted their result and it didn't match their own graph (but maybe I made a typo)
 
Last edited by a moderator:
Top